Diễn Đàn MathScopeDiễn Đàn MathScope
  Diễn Đàn MathScope
Ghi Danh Hỏi/Ðáp Community Lịch

Go Back   Diễn Đàn MathScope > Sơ Cấp > Đại Số và Lượng Giác > Chuyên Đề

News & Announcements

Ngoài một số quy định đã được nêu trong phần Quy định của Ghi Danh , mọi người tranh thủ bỏ ra 5 phút để đọc thêm một số Quy định sau để khỏi bị treo nick ở MathScope nhé !

* Nội quy MathScope.Org

* Một số quy định chung !

* Quy định về việc viết bài trong diễn đàn MathScope

* Nếu bạn muốn gia nhập đội ngũ BQT thì vui lòng tham gia tại đây

* Những câu hỏi thường gặp

* Về việc viết bài trong Box Đại học và Sau đại học


Trả lời Gởi Ðề Tài Mới
 
Ðiều Chỉnh Xếp Bài
Old 16-02-2009, 03:55 PM   #1
Conan Edogawa
+Thành Viên+
 
Conan Edogawa's Avatar
 
Tham gia ngày: Sep 2008
Đến từ: Trường ĐH Kinh tế TP.HCM
Bài gởi: 397
Thanks: 136
Thanked 303 Times in 150 Posts
500 BĐT chọn lọc của Cao Minh Quang

các bạn giải giúp mình bài 151,158,209,427 trong tài liệu trên dùm mình.Lưu ý chỉ áp dụng BDT Cauchy,nếu ko ra mới dùng cách khác nha.Cảm ơn nhiều
[RIGHT][I][B]Nguồn: MathScope.ORG[/B][/I][/RIGHT]
 
File Kèm Theo
Kiểu File : pdf 500_Bat_dang_thuc_chon_loc_-_Cao_Minh_Quang_5561_35884677.pdf (644.8 KB, 1387 lần tải)
Conan Edogawa is offline   Trả Lời Với Trích Dẫn
The Following 5 Users Say Thank You to Conan Edogawa For This Useful Post:
Franky.eco (04-12-2009), huynhminhman96 (07-05-2012), JohnnyCage (04-06-2009), Math Long (17-03-2009), tienanh_tx (28-08-2012)
Old 18-02-2009, 12:14 PM   #2
Conan Edogawa
+Thành Viên+
 
Conan Edogawa's Avatar
 
Tham gia ngày: Sep 2008
Đến từ: Trường ĐH Kinh tế TP.HCM
Bài gởi: 397
Thanks: 136
Thanked 303 Times in 150 Posts
giải giúp mình 4 bài trên đi các bạn,cảm ơn nhiều
[RIGHT][I][B]Nguồn: MathScope.ORG[/B][/I][/RIGHT]
 
Conan Edogawa is offline   Trả Lời Với Trích Dẫn
Old 20-02-2009, 09:47 AM   #3
k30101201
+Thành Viên+
 
k30101201's Avatar
 
Tham gia ngày: Sep 2008
Bài gởi: 44
Thanks: 4
Thanked 8 Times in 8 Posts
Cho $a,b,c \in (0,1) $.CMR $\sqrt{abc}+\sqrt{(1-a)(1-b)(1-c)}<1 $.

Dễ dàng chứng minh được mọi $x \in (0,1) $ thì $x^{\frac{1}{2}}<x^{\frac{1}{3}} $.
Áp dụng cho ta $\sqrt{abc}<\sqrt[3]{abc}<\frac{a+b+c}{3} $ và $\sqrt{(1-a)(1-b)(1-c)}<\sqrt[3]{(1-a)(1-b)(1-c)}<\frac{3-a-b-c}{3} $.
Cộng lại ta có đpcm
==============
Cho $a,b,c $ là các số thực dương thỏa $abc=1 $.CMR $\frac{b+c}{\sqrt a}+\frac{c+a}{\sqrt b}+\frac{a+b}{\sqrt c}\geq \sqrt a+\sqrt b +\sqrt c+3 $.
Ta có $VT \geq 2(\sqrt{\frac{bc}{a}}+\sqrt{\frac{ca}{b}} $$+\sqrt{\frac{ab}{c}}) $.
Tách ra áp dụng AM-GM la okie
[RIGHT][I][B]Nguồn: MathScope.ORG[/B][/I][/RIGHT]
 
__________________
Math + Linux + Web

thay đổi nội dung bởi: k30101201, 20-02-2009 lúc 10:14 AM Lý do: Tự động gộp bài
k30101201 is offline   Trả Lời Với Trích Dẫn
The Following User Says Thank You to k30101201 For This Useful Post:
CONAN (04-04-2009)
Old 20-02-2009, 04:56 PM   #4
Conan Edogawa
+Thành Viên+
 
Conan Edogawa's Avatar
 
Tham gia ngày: Sep 2008
Đến từ: Trường ĐH Kinh tế TP.HCM
Bài gởi: 397
Thanks: 136
Thanked 303 Times in 150 Posts
xin mọi người giải giúp mình bài 151,158,209,427 trong tài liệu trên,cảm ơn nhiều
[RIGHT][I][B]Nguồn: MathScope.ORG[/B][/I][/RIGHT]
 
Conan Edogawa is offline   Trả Lời Với Trích Dẫn
Old 03-03-2009, 11:35 AM   #5
Conan Edogawa
+Thành Viên+
 
Conan Edogawa's Avatar
 
Tham gia ngày: Sep 2008
Đến từ: Trường ĐH Kinh tế TP.HCM
Bài gởi: 397
Thanks: 136
Thanked 303 Times in 150 Posts
trời ơi sao lâu wa rồi mà vẫn chưa ai giải giúp em vậy,mong mọi người nhanh dùm,cảm ơn nhiều
[RIGHT][I][B]Nguồn: MathScope.ORG[/B][/I][/RIGHT]
 
Conan Edogawa is offline   Trả Lời Với Trích Dẫn
Old 04-03-2009, 07:14 AM   #6
dung_thanh
+Thành Viên+
 
Tham gia ngày: Feb 2009
Bài gởi: 22
Thanks: 3
Thanked 4 Times in 3 Posts
Trích:
Nguyên văn bởi Conan Edogawa View Post
trời ơi sao lâu wa rồi mà vẫn chưa ai giải giúp em vậy,mong mọi người nhanh dùm,cảm ơn nhiều
Bài 158: Giải bằng Holder nhưng thực chất cũng là bất đẳng thức cauchy.Vì BDt nào giải dc bằng holder cũng giải dc bằng cauchy.
$V{T^3} = {\left( {\sum {\sqrt[3]{{\frac{{1 + 6ab}}{a}}}} } \right)^3} = {\left( {\sum {\sqrt[3]{{\frac{{bc(1 + 6ab)}}{{abc}}}}} } \right)^3} \le (bc + ca + ab)(3 + 6(ab + bc + ca)).\frac{3}{{abc}} = \frac{{27}}{{abc}} $
mà theo BDT cauchy thì $ab.bc.ca \le {\left( {\frac{{ab + ca + ca}}{3}} \right)^3} \Rightarrow \frac{{27}}{{abc}} \le \frac{1}{{{a^3}{b^3}{c^3}}} $
từ đó ta có đpcm. umb:
[RIGHT][I][B]Nguồn: MathScope.ORG[/B][/I][/RIGHT]
 
dung_thanh is offline   Trả Lời Với Trích Dẫn
The Following 2 Users Say Thank You to dung_thanh For This Useful Post:
CONAN (04-04-2009), Conan Edogawa (04-03-2009)
Old 04-03-2009, 04:18 PM   #7
Minh Tuấn
+Thành Viên Danh Dự+
 
Tham gia ngày: Jul 2008
Bài gởi: 218
Thanks: 13
Thanked 78 Times in 41 Posts
Trích:
Nguyên văn bởi Conan Edogawa View Post
các bạn giải giúp mình bài 151,158,209,427 trong tài liệu trên dùm mình.Lưu ý chỉ áp dụng BDT Cauchy,nếu ko ra mới dùng cách khác nha.Cảm ơn nhiều
Bài 151: Ta có:
$VT\leq \frac{xyz(\sqrt{3(x^2+y^2+z^2)}+\sqrt{x^2+y^2+z^2} )}{(x^2+y^2+z^2)(xy+yz+zx)}=\frac{(\sqrt{3}+1)xyz} {\sqrt{x^2+y^2+z^2}(xy+yz+zx)}\leq 1 $
Vì:
$\sqrt{x^2+y^2+z^2}\geq \sqrt{3\sqrt[3]{x^2y^2z^2}}=\sqrt{3}\sqrt[3]{xyz} $
$xy+yz+zx\geq 3\sqrt[3]{x^2y^2z^2} $
Bài 158:
Áp dụng BĐT Holder ta có:
$VT^3\leq (1+6ab+1+6bc+1+6ca)(\frac{1}{a}+\frac{1}{b}+\frac{ 1}{c})(1+1+1)=\frac{27}{abc}=\frac{27ab.bc.ca}{a^3 b^3c^3}\leq \frac{1}{(abc)^3} $
Bài 209:
Mũ ba cả 2 vế rồi rút gọn ta được BĐT cần chứng minh tương đương với:
$9(abc)^2(1+6abc(a+b+c))\leq 1 $
Lại có:
$(ab.bc.ca)\leq (\frac{ab+bc+ca}{3})^3=\frac{1}{3} $
và
$1+6(ab.bc+bc.ca+ca.ab)\leq 1+6\frac{(ab+bc+ca)^2}{3}=3 $
Suy ra đpcm
Bài 427:
Ta có:
$\frac{a^2}{b}+9a^2b\geq 2\sqrt{\frac{a^2}{b}9a^2b}=6a^2 $
Thiết lập 2 BĐT tương tự suy ra:
$\frac{a^2}{b}+\frac{b^2}{c}+\frac{c^2}{a}+9(a^2b+b ^2c+c^2a)\geq 6(a^2+b^2+c^2) $
lại có: $a^2+b^2+c^2\geq 3(a^2b+b^2c+c^2a) $
Thật vậy BĐT tương đương với:
$a^2(a-2b+c)+b^2(a+b-2c)+c^2(-2a+b+c) \geq0 (*) $
(*) đúng vì $a^3+ab^2\geq2a^2b,... $
Vậy: $\frac{a^2}{b}+\frac{b^2}{c}+\frac{c^2}{a}+9(a^2b+b ^2c+|c^2a)\geq 3(a^2+b^2+c^2)+3(a^2+b^2+c^2) \geq3(a^2+b^2+c^2)+ 9(a^2b+b^2c+c^2a) $
Suy ra đpcm
[RIGHT][I][B]Nguồn: MathScope.ORG[/B][/I][/RIGHT]
 
Minh Tuấn is offline   Trả Lời Với Trích Dẫn
The Following 4 Users Say Thank You to Minh Tuấn For This Useful Post:
CONAN (04-04-2009), Conan Edogawa (05-03-2009), je.triste (03-08-2011), Unknowing (19-01-2011)
Old 05-03-2009, 10:19 PM   #8
Conan Edogawa
+Thành Viên+
 
Conan Edogawa's Avatar
 
Tham gia ngày: Sep 2008
Đến từ: Trường ĐH Kinh tế TP.HCM
Bài gởi: 397
Thanks: 136
Thanked 303 Times in 150 Posts
làm dùm em thêm bài 399 đi anh Minh Tuấn,cũng chỉ = Cauchy thoi nha anh
[RIGHT][I][B]Nguồn: MathScope.ORG[/B][/I][/RIGHT]
 
Conan Edogawa is offline   Trả Lời Với Trích Dẫn
Old 09-04-2009, 10:19 AM   #9
CONAN
+Thành Viên+
 
Tham gia ngày: Apr 2009
Bài gởi: 6
Thanks: 17
Thanked 2 Times in 1 Post
nhờ bạn gửi file bài giải cuốn sách này cho mình được ko.
[RIGHT][I][B]Nguồn: MathScope.ORG[/B][/I][/RIGHT]
 
CONAN is offline   Trả Lời Với Trích Dẫn
Old 13-04-2009, 05:23 PM   #10
Conan Edogawa
+Thành Viên+
 
Conan Edogawa's Avatar
 
Tham gia ngày: Sep 2008
Đến từ: Trường ĐH Kinh tế TP.HCM
Bài gởi: 397
Thanks: 136
Thanked 303 Times in 150 Posts
nếu mình có bài giải rồi thì việc gì mình phải lên đây nhờ các cao thủ giải giúp nữa
[RIGHT][I][B]Nguồn: MathScope.ORG[/B][/I][/RIGHT]
 
Conan Edogawa is offline   Trả Lời Với Trích Dẫn
Old 07-11-2009, 12:10 AM   #11
Pro LHP
+Thành Viên+
 
Tham gia ngày: Nov 2009
Bài gởi: 1
Thanks: 0
Thanked 0 Times in 0 Posts
Chào bạn

Trích:
Nguyên văn bởi Conan Edogawa View Post
nếu mình có bài giải rồi thì việc gì mình phải lên đây nhờ các cao thủ giải giúp nữa
nghe noi đây là cuốn nóng bỏng trên thị trường fai ko
[RIGHT][I][B]Nguồn: MathScope.ORG[/B][/I][/RIGHT]
 
Pro LHP is offline   Trả Lời Với Trích Dẫn
Old 07-02-2010, 11:28 AM   #12
Conan Edogawa
+Thành Viên+
 
Conan Edogawa's Avatar
 
Tham gia ngày: Sep 2008
Đến từ: Trường ĐH Kinh tế TP.HCM
Bài gởi: 397
Thanks: 136
Thanked 303 Times in 150 Posts
cuốn này bài tập được mà tiếc ko có đáp án => cảm nhận riêng là ko hay lắm
[RIGHT][I][B]Nguồn: MathScope.ORG[/B][/I][/RIGHT]
 
Conan Edogawa is offline   Trả Lời Với Trích Dẫn
Old 02-08-2011, 09:37 PM   #13
assassin_dark
+Thành Viên+
 
Tham gia ngày: Aug 2011
Bài gởi: 1
Thanks: 0
Thanked 0 Times in 0 Posts
Anh chị nào giúp em giải bài 393 với :
Cho a,b,c là các số dương .Chứng minh rằng

[RIGHT][I][B]Nguồn: MathScope.ORG[/B][/I][/RIGHT]
 
assassin_dark is offline   Trả Lời Với Trích Dẫn
Trả lời Gởi Ðề Tài Mới

Bookmarks


Quuyền Hạn Của Bạn
You may not post new threads
You may not post replies
You may not post attachments
You may not edit your posts

BB code is Mở
Smilies đang Mở
[IMG] đang Mở
HTML đang Tắt

Chuyển đến


Múi giờ GMT. Hiện tại là 10:15 PM.


Powered by: vBulletin Copyright ©2000-2024, Jelsoft Enterprises Ltd.
Inactive Reminders By mathscope.org
[page compression: 86.31 k/100.19 k (13.86%)]